Question on the notaion used to define Lie Derviative

  • Thread starter Thread starter logarithmic
  • Start date Start date
Click For Summary
SUMMARY

The discussion centers on the notation used in the definition of the Lie derivative, specifically the term Y_{\theta_t(p)}. Y represents a vector field, while \theta_t(p) is a function that indicates the evaluation point of the vector field. The consensus confirms that Y_{\theta_t(p)} indeed means to evaluate the vector field Y at the point specified by \theta_t(p), aligning with the notation presented in the referenced article on PlanetMath.

PREREQUISITES
  • Understanding of vector fields in differential geometry
  • Familiarity with the concept of the Lie derivative
  • Knowledge of the notation used in mathematical analysis
  • Basic grasp of functions and their evaluations in mathematical contexts
NEXT STEPS
  • Study the properties and applications of the Lie derivative in differential geometry
  • Explore vector fields and their significance in physics and mathematics
  • Review advanced mathematical notation and its implications in various contexts
  • Investigate the relationship between vector fields and flows in dynamical systems
USEFUL FOR

Mathematicians, physicists, and students of differential geometry seeking to deepen their understanding of the Lie derivative and its notation.

logarithmic
Messages
103
Reaction score
0
I have a definition of the Lie derivative, that is the one found here: http://planetmath.org/encyclopedia/LieDerivative2.html

However, I'm not sure what the notation Y_{\theta_t(p)} used in that article means.

Y is a vector field and \theta_t(p) is a function. Does it mean evaluate Y at \theta_t(p)}[/tex]?

Can someone explain.
 
Last edited by a moderator:
Physics news on Phys.org
logarithmic said:
Does it mean evaluate Y at \theta_t(p)}?

Yes. Your guess is good and it is consistent with the notation used throughout this article.
 
Question: A clock's minute hand has length 4 and its hour hand has length 3. What is the distance between the tips at the moment when it is increasing most rapidly?(Putnam Exam Question) Answer: Making assumption that both the hands moves at constant angular velocities, the answer is ## \sqrt{7} .## But don't you think this assumption is somewhat doubtful and wrong?

Similar threads

  • · Replies 8 ·
Replies
8
Views
2K
  • · Replies 7 ·
Replies
7
Views
2K
  • · Replies 22 ·
Replies
22
Views
6K
Replies
14
Views
2K
  • · Replies 20 ·
Replies
20
Views
3K
  • · Replies 2 ·
Replies
2
Views
2K
  • · Replies 3 ·
Replies
3
Views
2K
  • · Replies 10 ·
Replies
10
Views
2K
Replies
1
Views
3K
  • · Replies 7 ·
Replies
7
Views
2K